Answer:
Orthocentre (intersection of altitudes) is at (37/10, 19/5)
Step-by-step explanation:
Given three vertices of a triangle
A(-5, -2)
B(7, -5)
C(3, 1)
Solution A by geometry
Slope AB = (yb-ya) / (xb-xa) = (-5-(-2)) / (7-(-5)) = -3/12 = -1/4
Slope of line normal to AB, nab = -1/(-1/4) = 4
Altitude of AB = line through C normal to AB
(y-yc) = nab(x-xc)
y-1 = (4)(x-3)
y = 4x-11 .........................(1)
Slope BC = (yc-yb) / (xc-yb) = (1-(-5) / (3-7)= 6 / (-4) = -3/2
Slope of line normal to BC, nbc = -1 / (-3/2) = 2/3
Altitude of BC
(y-ya) = nbc(x-xa)
y-(-2) = (2/3)(x-(-5)
y = 2x/3 + 10/3 - 2
y = (2/3)(x+2) ........................(2)
Orthocentre is at the intersection of (1) & (2)
Equate right-hand sides
4x-11 = (2/3)(x+2)
Cross multiply and simplify
12x-33 = 2x+4
10x = 37
x = 37/10 ...................(3)
substitute (3) in (2)
y = (2/3)(37/10+2)
y=(2/3)(57/10)
y = 19/5 ......................(4)
Therefore the orthocentre is at (37/10, 19/5)
Alternative Solution B using vectors
Let the position vectors of the vertices represented by
a = <-5, -2>
b = <7, -5>
c = <3, 1>
and the position vector of the orthocentre, to be found
d = <x,y>
the line perpendicular to BC through A
(a-d).(b-c) = 0 "." is the dot product
expanding
<-5-x,-2-y>.<4,-6> = 0
simplifying
6y-4x-8 = 0 ...................(5)
Similarly, line perpendicular to CA through B
<b-d>.<c-a> = 0
<7-x,-5-y>.<8,3> = 0
Expand and simplify
-3y-8x+41 = 0 ..............(6)
Solve for x, (5) + 2(6)
-20x + 74 = 0
x = 37/10 .............(7)
Substitute (7) in (6)
-3y - 8(37/10) + 41 =0
3y = 114/10
y = 19/5 .............(8)
So orthocentre is at (37/10, 19/5) as in part A.
Assume that a sample is used to estimate a population proportion p. Find the margin of error E that corresponds to the given statistics and confidence level. Round the margin of error to four decimal places. 95% confidence; n = 349, x = 42
Answer:
0.5705Step-by-step explanation:
Margin of error is expressed as M.E = [tex]z * \sqrt{\frac{\sigma}{n} }[/tex] where;
z is the z score at 95% confidence
[tex]\sigma[/tex] is the standard deviation
n is the sample size
Given n = 349, [tex]\sigma = 42[/tex] and z score at 95% confidence = 1.645
Substituting this values into the formula above we will have;
M.E = [tex]1.645*\sqrt{\frac{42}{349} }[/tex]
[tex]M.E = 1.645*\sqrt{0.1203} \\\\M.E = 1.645*0.3468\\\\M.E = 0.5705 (to\ four\ dp)[/tex]
The original price of a burrito was $8. The price was increased by 10%. What is the new price?
Answer:
The new price is 8.80
Step-by-step explanation:
First find the increase
8 * 10%
8 * .10
.80
Add this to the original price
8 + .80
8.80
The new price is 8.80
Answer:
the answer is $8.80 :))
The graph shows government receipts and outlays (both on-budget and off-budget) for . In what years did receipts appear to climb faster than outlays?
Answer: Receipts appear to climb faster than outlays in 2001, 2002, 2003 and 2004
Step-by-step explanation: So if you see the graph you can see that receipts started to climb faster by the middle of 2001 so in conclusion Receipts appear to climb faster than outlays in 2001, 2002, 2003 and 2004 hope that help you
When government revenues exceed government outlays in a particular year, this is called a d) budget surplus.
What is budget surplus?A budget surplus occurs when a government's income from taxes and other sources of revenue exceed its expenses and spending on public programs, services, and infrastructure. A budget surplus is a positive indicator for the government's financial health as it indicates that the government is able to manage its finances well, generate more revenue than it spends, and potentially pay off any debts.
Here, we have,
In contrast, a budget deficit occurs when government spending exceeds its revenue, resulting in increased debt and a potentially negative impact on the government's credit rating.
Budget surpluses can be used to invest in public projects or programs, reduce debt, or returned to taxpayers in the form of tax cuts.
However, it is important to note that budget surpluses can also be a result of reduced spending on public programs, which can have a negative impact on social services and infrastructure.
Therefore, the correct answer is d) budget surplus.
Learn more about budget surplus here:
brainly.com/question/30453443
#SPJ2
coomplete question:
When government revenues exceed government outlays in a particular year, this is called
A.
the national debt.
B.
a budget deficit.
C.
fiscal policy.
D.
a budget surplus.
If y varies directly as x, and y is 6 when x is 72, what is the value of y when x is 8?
NO
54
оо
96
Answer:
2/3
Step-by-step explanation:
The equation for direct variation is: y = kx, where k is a constant.
Here, we see that y varies directly with x when y = 6 and x = 72, so let's plug these values into the formula to find k:
y = kx
6 = k * 72
k = 6/72 = 1/12
So, k = 1/12. Now our formula is y = (1/12)x. Plug in 8 for x to find y:
y = (1/12)x
y = (1/12) * 8 = 8/12 = 2/3
Thus, y = 2/3.
~ an aesthetics lover
Answer:
Step-by-step explanation: I hope i'm right
[tex]y \alpha x\\y=kx....(1)\\6=72k\\\frac{6}{72} =\frac{72k}{72} \\\\1/12 =k\\y = 1/12x=relationship-between;x-and;y\\x =8 , y =?\\y = \frac{8}{12} \\Cross-Multiply\\12y =8\\12y/12 = 8/12\\\\y = 2/3[/tex]
i neeed help thanksssss
Answer:
Volume: 366.6
Surface Area: 314.2
Explanation (look at below)
Step-by-step explanation:
Volume:
The radius of this sphere is 5 (half of 10). The equation will be [tex]\frac{4}{3} \pi[/tex]5^3
When you calculate that, it will become: 366.6<-- rounded to the nearest tenth.
Surface Area:
4[tex]\pi[/tex]5^2
=100Π
=314.2<-- rounded to the nearest tenth.
314.2 is the nearest tenth digit no.
The area of an Equilateral triangle is given by the formula A= 3pi squared/4(s)Squared. Which formula represents the length of equilateral triangle’s side S?
Answer:
The formula that represents the length of an equilateral triangle’s side (s) in terms of the triangle's area (A) is [tex]\text{s}= \sqrt{ \frac{4 \text{A}}{\sqrt{3} }}[/tex] .
Step-by-step explanation:
We are given the area of an Equilateral triangle which is A = [tex]\frac{\sqrt{3} }{4} \times \text{s}^{2}[/tex] . And we have to represent the length of an equilateral triangle’s side (s) in terms of the triangle's area (A).
So, the area of an equilateral triangle = [tex]\frac{\sqrt{3} }{4} \times \text{s}^{2}[/tex]
where, s = side of an equilateral triangle
A = [tex]\frac{\sqrt{3} }{4} \times \text{s}^{2}[/tex]
Cross multiplying the fractions we get;
[tex]4 \times A = \sqrt{3} \times \text{s}^{2}[/tex]
[tex]\sqrt{3} \times \text{s}^{2}= 4\text{A}[/tex]
Now. moving [tex]\sqrt{3}[/tex] to the right side of the equation;
[tex]\text{s}^{2}= \frac{4 \text{A}}{\sqrt{3} }[/tex]
Taking square root both sides we get;
[tex]\sqrt{\text{s}^{2}} = \sqrt{ \frac{4 \text{A}}{\sqrt{3} }}[/tex]
[tex]\text{s}= \sqrt{ \frac{4 \text{A}}{\sqrt{3} }}[/tex]
Hence, this formula represents the length of an equilateral triangle’s side (s) in terms of the triangle's area (A).
Look at the parallelogram ABCD shown below. The table below shows the steps to prove that if the quadrilateral ABCD is a parallelogram, then its opposite sides are congruent. Statement Reasons 1 . AB is parallel to DC and AD is parallel to BC - definition of parallelogram 2 . angle 1 = angle 2, angle 3 = angle 4 - if two parallel lines are cut by a transversal then the corresponding angles are congruent 3 . BD = BD - Reflexive Property 4 . triangles ADB and CBD are congruent - if two sides and the included angle of a triangle are congruent to the corresponding sides and angle of another triangle , then the triangles are congruent by SAS postulate 5 . AB = DC, AD = BC - corresponding parts of congruent triangles are congruent Which statement is true about the table? 1. It is not correct because it provides incorrect sequence of statement 2 and statement 4. 2. It is not correct because it does not provide correct reasons for statement 2 and statement 4. 3. It is accurate because it provides the correct sequence of statements. 4. It is accurate because it provides the correct reasons for the statements.
Answer:
2
Step-by-step explanation:
Need help with this as soon as possible.
Answer:
1, 2
Step-by-step explanation:
The integers, x, that satisfy 1 <= x <= 8 are
1, 2, 3, 4, 5, 6, 7, 8
Now we solve the inequality for x.
-x + 4 >= 2
-x >= -2
x <= 2
Now we see which of the eight integers above make the inequality true.
Only 1 and 2 satisfy the inequality.
2. Salvador has 10 cards, each with one number on
it. The numbers are 2, 3, 4,5,5,7,7,7,7,7.
Salvador is going to make a row containing all 10
cards. How many ways can he order the row?
Answer:
15,120 number of ways.Step-by-step explanation:
This is a permutation problem. Given the 10 cards with numbers 2, 3, 4,5,5,7,7,7,7,7 on it, if Salvador is going to make a row call, the number of ways he can order a row is as shown below;
Total number of cards = 10!
number of times the digit 5 was repeated = 2times
number of times the digit 7 was repeated = 5times
The number of ways he can make a row call = 10!/2!5!
= 10*9*8*7*6*5!/2*5!
= 10*9*8*7*6/2
= 10*9*8*7*3
= 15,120 different ways
Hence, the number of ways he can order the row is 15,120 number of ways.
Find the value of annuity if the periodic deposit is $400 at 4% compounded monthly for 18 years
Answer:
~820.8$
Step-by-step explanation:
The total money (M) after 18 years could be calculated by:
M = principal x (1 + rate)^time
with
principal = 400$
rate = 4% compounded monthly = 0.04/12
time = 18 years = 18 x 12 = 216 months (because of compounded monthly rate)
=> M = 400 x (1 + 0.04/12)^216 = ~820.8$
How to do this? what is the answer??
Answer:
I think that is the C
Step-by-step explanation:
Answer:
Option B is the correct answer.
Step-by-step explanation:
here, arc RT =162°
as in question given that the value of arc RT is 162° the value of angle RST is 1/2 of 162°.
so, its value must be 81°only.
hope it helps..
Which expression is equivalent to 2m^2 - m^2(7-m)+6m^2?
Answer:
[tex]m^3+m^2[/tex]
Step-by-step explanation:
=> [tex]2m^2-m^2(7-m)+6m^2[/tex]
Collecting like terms and expanding the brackets
=> [tex]2m^2+6m^2-7m^2+m^3[/tex]
=> [tex]8m^2-7m^2+m^3[/tex]
=> [tex]m^2+m^3[/tex]
=> [tex]m^3+m^2[/tex]
Suppose that E and F are two events and that P(E and F) = 0.2 and P(E) = 0.4. What is P(F/E)
Answer:
The conditional probability is given by
P(F|E) = P(E and F)/P(E)
P(F|E) = 0.2/0.4
P(F|E) = 0.5
P(F|E) = 50%
Step-by-step explanation:
Recall that the conditional probability is given by
∵ P(B | A) = P(A and B)/P(A)
For the given case,
P(F|E) = P(E and F)/P(E)
Where P(F|E) is the probability of event F occurring given that event E has occurred.
The probability of event E and F is given as
P(E and F) = 0.2
The probability of event E is given as
P(E) = 0.4
So, the conditional probability is
P(F|E) = P(E and F)/P(E)
P(F|E) = 0.2/0.4
P(F|E) = 0.5
P(F|E) = 50%
If C(x) is the cost of producing x units of a commodity, then the average cost per unit is c(x) = C(x)/x. Consider the cost function C(x) given below. C(x) = 54,000 + 130x + 4x3/2 (a) Find the total cost at a production level of 1000 units. (Round your answer to the nearest cent.) $ (b) Find the average cost at a production level of 1000 units. (Round your answer to the nearest cent.) $ per unit (c) Find the marginal cost at a production level of 1000 units. (Round your answer to the nearest cent.) $ per unit (d) Find the production level that will minimize the average cost. (Round your answer to the nearest whole number.) units (e) What is the minimum average cost? (Round your answer to the nearest dollar.) $ per unit
Answer:
Step-by-step explanation:
Given that:
If C(x) = the cost of producing x units of a commodity
Then;
then the average cost per unit is c(x) = [tex]\dfrac{C(x)}{x}[/tex]
We are to consider a given function:
[tex]C(x) = 54,000 + 130x + 4x^{3/2}[/tex]
And the objectives are to determine the following:
a) the total cost at a production level of 1000 units.
So;
If C(1000) = the cost of producing 1000 units of a commodity
[tex]C(1000) = 54,000 + 130(1000) + 4(1000)^{3/2}[/tex]
[tex]C(1000) = 54,000 + 130000 + 4( \sqrt[2]{1000^3} )[/tex]
[tex]C(1000) = 54,000 + 130000 + 4(31622.7766)[/tex]
[tex]C(1000) = 54,000 + 130000 + 126491.1064[/tex]
[tex]C(1000) = $310491.1064[/tex]
[tex]\mathbf{C(1000) \approx $310491.11 }[/tex]
(b) Find the average cost at a production level of 1000 units.
Recall that :
the average cost per unit is c(x) = [tex]\dfrac{C(x)}{x}[/tex]
SO;
[tex]c(x) =\dfrac{(54,000 + 130x + 4x^{3/2})}{x}[/tex]
Using the law of indices
[tex]c(x) =\dfrac{54000}{x} + 130 + 4x^{1/2}[/tex]
[tex]c(1000) = \dfrac{54000}{1000}+ 130 + {4(1000)^{1/2}}[/tex]
c(1000) =$ 310.49 per unit
(c) Find the marginal cost at a production level of 1000 units.
The marginal cost is C'(x)
Differentiating C(x) = 54,000 + 130x + 4x^{3/2} to get C'(x) ; we Have:
[tex]C'(x) = 0 + 130 + 4 \times \dfrac{3}{2} \ x^{\dfrac{3}{2}-1}[/tex]
[tex]C'(x) = 0 + 130 + 2 \times \ {3} \ x^{\frac{1}{2}}[/tex]
[tex]C'(x) = 0 + 130 + \ {6}\ x^{\frac{1}{2}}[/tex]
[tex]C'(1000) = 0 + 130 + \ {6} \ (1000)^{\frac{1}{2}}[/tex]
[tex]C'(1000) = 319.7366596[/tex]
[tex]\mathbf{C'(1000) = \$319.74 \ per \ unit}[/tex]
(d) Find the production level that will minimize the average cost.
the average cost per unit is c(x) = [tex]\dfrac{C(x)}{x}[/tex]
[tex]c(x) =\dfrac{54000}{x} + 130 + 4x^{1/2}[/tex]
the production level that will minimize the average cost is c'(x)
differentiating [tex]c(x) =\dfrac{54000}{x} + 130 + 4x^{1/2}[/tex] to get c'(x); we have
[tex]c'(x)= \dfrac{54000}{x^2} + 0+ \dfrac{4}{2 \sqrt{x} }[/tex]
[tex]c'(x)= \dfrac{54000}{x^2} + 0+ \dfrac{2}{ \sqrt{x} }[/tex]
Also
[tex]c''(x)= \dfrac{108000}{x^3} -x^{-3/2}[/tex]
[tex]c'(x)= \dfrac{54000}{x^2} + \dfrac{4}{2 \sqrt{x} } = 0[/tex]
[tex]x^2 = 27000\sqrt{x}[/tex]
[tex]\sqrt{x} (x^{3/2} - 27000) =0[/tex]
x= 0; or [tex]x= (27000)^{2/3}[/tex] = [tex]\sqrt[3]{27000^2}[/tex] = 30² = 900
Since production cost can never be zero; then the production cost = 900 units
(e) What is the minimum average cost?
the minimum average cost of c(900) is
[tex]c(900) =\dfrac{54000}{900} + 130 + 4(900)^{1/2}[/tex]
c(900) = 60 + 130 + 4(30)
c(900) = 60 +130 + 120
c(900) = $310 per unit
Helppppppp pleaseeee
Answer:
d 13
Step-by-step explanation:
Nitrates are groundwater contaminants derived from fertilizer, septic tank seepage and other sewage. Nitrate poisoning is particularly hazardous to infants under the age of 6 months. The maximum contaminant level (MCL) is the highest level of a contaminant the government allows in drinking water. For nitrates, the MCL is 10mg/L. The health department wants to know what proportion of wells in Madison Count that have nitrate levels above the MCL. A worker has been assigned to take a simple random sample of wells in the county, measure the nitrate levels, and assess compliance. What sample size should the health department obtain if the estimate is desired to be within 2% with 95% confidence if: (hint: there are two different methods)There is no prior information available?
Answer:
The sample size is [tex]n = 2401[/tex]
Step-by-step explanation:
From the question we are told that
The margin of error is [tex]E = 0.02[/tex]
Given that the confidence level is 95% then the level of significance can be mathematically evaluated as
[tex]\alpha = 100 - 95[/tex]
[tex]\alpha = 5\%[/tex]
[tex]\alpha = 0.05[/tex]
Next we would obtain the critical value of [tex]\frac{\alpha }{2}[/tex] from the z-table , the values is
[tex]Z_{\frac{\alpha }{2} } = Z_{\frac{0.05 }{2} } = 1.96[/tex]
The reason we are obtaining critical value of [tex]\frac{\alpha }{2}[/tex] instead of [tex]\alpha[/tex] is because
[tex]\alpha[/tex] represents the area under the normal curve where the confidence level interval ( [tex]1-\alpha[/tex] ) did not cover which include both the left and right tail while
[tex]\frac{\alpha }{2}[/tex] is just the area of one tail which what we required to calculate the sample size
NOTE: We can also obtain the value using critical value calculator (math dot armstrong dot edu)
Generally the sample size is mathematically evaluated as
[tex]n = [ \frac{Z_{\frac{\alpha }{2} }}{E} ]^2 * \r p (1- \r p)[/tex]
Where [tex]\r p[/tex] is the proportion of sample taken which we will assume to be [tex]\r p = 0.5[/tex]
substituting values
[tex]n = [\frac{ 1.96}{0.02} ]^2 *( 0.5 (1- 0.5)[/tex]
[tex]n = 2401[/tex]
Please help!!
Find the value of x.
X=
Answer:
Step-by-step explanation:
Hello,
We can write three equations thanks to Pythagoras
[tex]AB^2+AC^2=(7+3)^2\\x^2+7^2=AB^2\\x^2+3^2=BC^2\\[/tex]
So it comes
[tex]x^2+7^2+x^2+3^2=(7+3)^2\\\\2x^2=100-49-9=42\\\\x^2 = 42/2=21\\\\x = \sqrt{\boxed{21}}\\[/tex]
Hope this helps
Answer:
x = [tex]\sqrt{21}[/tex]
Step-by-step explanation:
Δ BCD and Δ ABD are similar thus the ratios of corresponding sides are equal, that is
[tex]\frac{BD}{AD}[/tex] = [tex]\frac{CD}{BD}[/tex] , substitute values
[tex]\frac{x}{7}[/tex] = [tex]\frac{3}{x}[/tex] ( cross- multiply )
x² = 21 ( take the square root of both sides )
x = [tex]\sqrt{21}[/tex]
What single transformation maps Triangle ABC onto A’B’C’
Answer:
Your answer is B
Step-by-step explanation:
rotating about/around the origin taking a shape and rotating it with the same values but around the point (0,0). so rotating your shape ABC around (0,0) with the same value would give you the shape A'B'C'
A square matrix N is called nilpotent if there exists some positive integer k such that Nk = 0. Prove that if N is a nilpotent matrix, then the system Nx = 0 has nontrivial solutions.
Answer:
Nx = λx
Nx = 0, with x≠0
if N is nilpotent matrix, then the system Nx = 0 has non-trivial solutions
Step-by-step explanation:
given that
let N be a square matrix in order of n
note: N is nilpotent matrix with [tex]N^{k} = 0[/tex], k ∈ N
let λ be eigenvalue of N and let x be eigenvector corresponding to eigenvalue λ
Nx = λx (x≠0)
N²x = λNx = λ²x
∴[tex]N^{k}x[/tex] = (λ^k)x
[tex]N^{k}[/tex] = 0, (λ^k)x = [tex]0_{n}[/tex], where n is dimensional vector
where x = 0, (λ^k) = 0
λ = 0
therefore, Nx = λx
Nx = 0, with x≠0
note: if N is nilpotent matrix, then the system Nx = 0 has non-trivial solution
ASAP PLEASE HELP!!!!!! Find the y-intercept of the rational function. A rational function is graphed in the first quadrant, and in the second, third and fourth quadrants are other pieces of the graph. The graph crosses the x axis at negative 10 and crosses the y axis at negative 2.
Answer:
(0,-2)
Step-by-step explanation:
The y-intercept is simply when the function touches or crosses the y-axis.
We're told that the graph crosses the y-axis at -2. In other words, the y-intercept is at -2.
The ordered pair would be (0,-2)
Which of the following functions is graphed below?
Answer:
B.
Step-by-step explanation:
The function above the x-axis looks like a parabola, so it must have an x^2 term. It also has an open circle, so it must have a > symbol.
The function below the x-axis has a closed circle, so it must have a <= symbol. It is also a 3rd degree polynomial.
Answer: B.
Let T:V→W be a linear transformation from a vector space V into a vector space W. Prove that the range of T is a subspace of W.
Answer:
The range of T is a subspace of W.
Step-by-step explanation:
we have T:V→W
This is a linear transformation from V to W
we are required to prove that the range of T is a subspace of W
0 is a vector in range , u and v are two vectors in range T
T = T(V) = {T(v)║v∈V}
{w∈W≡v∈V such that T(w) = V}
T(0) = T(0ⁿ)
0 is Zero in V
0ⁿ is zero vector in W
T(V) is not an empty subset of W
w₁, w₂ ∈ T(v)
(v₁, v₂ ∈V)
from here we have that
T(v₁) = w₁
T(v₂) = w₂
t(v₁) + t(v₂) = w₁+w₂
v₁,v₂∈V
v₁+v₂∈V
with a scalar ∝
T(∝v) = ∝T(v)
such that
T(∝v) ∈T(v)
so we have that T(v) is a subspace of W. The range of T is a subspace of W.
witch term describes the point were the three medians of a triangle intersect
Answer:
The centroid
Step-by-step explanation:
Every triangle has exactly three medians, one from each vertex, and they all intersect each other at the triangle's centroid.
baby weights: a study was conducted to determin the average birth weight (in ounces) of babies born in hospitals in a five county area of a given state. A Simple Random Sample of Recent birth records at the local hospitals were selected and the confidence interval was calulated to be (117.89 ounces, 124.91), at a 95% level of confidence. Which statistic is appropriate for this confidences interval?
Answer: Sample mean [tex](\overline{x})[/tex]
Step-by-step explanation:
Given: A study was conducted to determine the average birth weight (in ounces) of babies born in hospitals in a five county area of a given state.
i.e. The parameter of the study is [tex]\mu[/tex] . (Population mean).
A Simple Random Sample of Recent birth records at the local hospitals were selected and the confidence interval was calculated to be (117.89 ounces, 124.91), at a 95% level of confidence.
Since a measure of sample is a statistic , and this case statistic is sample mean denoted by [tex]\overline{x}[/tex].
Hence, the statistic is appropriate for this confidences interval : [tex]\overline{x}[/tex]
When using the Distance Formula, the solution is the perimeter of a polygon.
true or false?
Answer:
false
Step-by-step explanation:
When solving the distance formula it is the distance from one point to another. If you had a rectangle and used the distance formula from each point then you would have a perimeter
Which inequality has -12 in its solution set?
A
B
С
D
X+6 <-8
X+42-6
X-3 >-10
X+55-4
ОА
B
D
Answer:
D) [tex]x+5\leq -4[/tex]
Step-by-step explanation:
We solve each of the inequalities
Option A
[tex]x+6<-8\\x<-8-6\\x<-14[/tex]
Option B
[tex]x+4\geq -6[/tex]
[tex]x\geq -6-4\\x\geq-10[/tex]
Option C
[tex]x-3>-10\\x>-10+3\\x>-7[/tex]
Option D
[tex]x+5\leq -4[/tex]
[tex]x\leq -4-5\\x\leq -9[/tex]
Therefore, only option D has -12 in its solution set.
Find the exact value of sin 5x/6
Answer:
1/2
Step-by-step explanation:
i think sin 5 pi/6
[tex]sin \frac{5 \pi}{6} =sin (\frac{\pi}{2} +\frac{\pi}{3} )=cos \frac{\pi}{3} =\frac{1}{2} \\or\\sin\frac{5 \pi}{6} =sin (\pi-\frac{\pi}{6} )=sin \frac{\pi}{6} =\frac{1}{2}[/tex]
The exact Value of sin 5x/6 is 1/2.
What is Trigonometry?The area of mathematics that deals with particular angles' functions and how to use those functions in calculations. There are six popular trigonometric functions for an angle. Sine (sin), cosine (cos), tangent (tan), cotangent (cot), secant (sec), and cosecant are their respective names and acronyms (csc).
Given:
sin 5x/6
We can Split it as
sin (5π / 6) = sin (π/2 + π/3)
= cos π/3
= 1/2
Also, sin (5π / 6) = sin (π - π/6)
= sin π/6
= 1/2
Hence, the exact value is 1/2.
Learn more about Trigonometry here:
https://brainly.com/question/29002217
#SPJ2
6th-grade math help me, please
Answer:
Question (2). Option (D)
Question (3). (a). 56
(b). 84
Step-by-step explanation:
Question (2).
Since, a% of b = [tex]\frac{a}{100}\times b[/tex]
42% of 350 = [tex]\frac{42}{100}\times 350[/tex]
Therefore, Option (d) is the correct option.
Question (3).
Total number of people who attended the music concert = 700
a). Percentage of people who arrived late in the concert = 8%
Therefore, number of people who attended the concert = 8% of 700
= [tex]\frac{8}{100}\times 700[/tex]
= 56
b). Percentage of people who bought the shirt = 12%
Number of people who bought the shirt = 12% of 700
= [tex]\frac{12}{100}\times 700[/tex]
= 84
helppppppppPPPPppppppPPPPppppPPpppppPPppPPpppPPPppp PLEASE do not look it up on any source
Answer:
open side up: 2%
closed side up: 10%
landing on side: 88%
Step-by-step explanation:
Jake tossed it 50 times, so to figure out probability it is easier to make the fraction over 100 so multiply the open side up by 2 the closed side up by 2 and the landing side up by 2 and make it over 100 then divide them.
The distance around a rectangular cafe is 35m . The ratio of length of the cafe to the width is 3:2. Find the dimension of the cafe
Hi there! :)
Answer:
Length = 10.5 m, width = 7 m.
Step-by-step explanation:
Given:
Perimeter, or P = 35 m
Ratio of l to w = 3 : 2
Since the ratio is 3 : 2, let l = 3x, and w = 2x.
We know that the formula for the perimeter of a rectangle is P = 2l + 2w. Therefore:
35 = 2(3x) + 2(2x)
35 = 6x + 4x
35 = 10x
x = 3.5
Plug this value of "x" into each expression to solve for the dimensions:
2(3.5) = w
w = 7 m
3(3.5) = l
l = 10.5 m
Therefore, the dimensions are:
Length = 10.5 m, width = 7 m.